Triangle A B C is cut by line segment S T. Line segment S T goes from side A B to side C B. Lines S T and A C are parallel. The length of S B is 10 feet, the length of B T is 9 feet, and the length of C T is 2.7 feet.
What is the length of Line segment S A?

1.89 ft
2.43 ft
3 ft
7 ft

Answers

Answer 1

In the triangle ABC, the length of the line segment SA=3 ft ( 3rd option).

In triangle ABC,

SB=10ftBT=9ftCT=2.7ft &ST|| AB

According to the triangle proportionality theory, we know that if a triangle has a line segment parallel to one of its sides, the line segment splits the other sides correspondingly.

Hence,

SA/SB=CT/BT

⇒SA/10=2.7/9

⇒SA=(2.7*10)/9

⇒SA=3

So, the length of SA is 3 ft.

Therefore the 3rd option is correct.

Learn more about the triangle proportionality theory

https://brainly.com/question/11807262?referrer=searchResults

#SPJ10

Triangle A B C Is Cut By Line Segment S T. Line Segment S T Goes From Side A B To Side C B. Lines S T

Related Questions

Find all solutions of the equation in the interval .
Write your answer in radians in terms of .
If there is more than one solution, separate them with commas.

Answers

The solutions to the trigonometric equation in the desired interval are given as follows:

[tex]\theta = \frac{\pi}{3}, \theta = \frac{5\pi}{3}[/tex]

What is the solution to the trigonometric equation?

The trigonometric equation is given by:

[tex]\sqrt{3}\cot{\theta} - 1 = 0[/tex]

Solving it similarly to an equation, we have that:

[tex]\sqrt{3}\cot{\theta} = 1[/tex]

[tex]\cot{\theta} = \frac{1}{\sqrt{3}}[/tex]

Since [tex]\cot{\theta} = \frac{1}{\tan{\theta}}[/tex], we have that the equation is equivalent to:

[tex]\tan{\theta} = \sqrt{3}[/tex]

The tangent is positive in the first and in the fourth quadrant. In the first quadrant, the angle [tex]\theta[/tex] with [tex]\tan{\theta} = \sqrt{3}[/tex] is:

[tex]\theta = \frac{\pi}{3}[/tex]

In the fourth quadrant, the equivalent angle is:

[tex]\theta = 2\pi - \frac{\pi}{3} = \frac{5\pi}{3}[/tex]

More can be learned about trigonometric equations at https://brainly.com/question/24680641

#SPJ1

Which of the following are solutions to the equation below?


4x^2 + 4x + 1 = 9


Check all that apply

Answers

Answer:

Option B and D

Step-by-step explanation:

Subtract 9 from both sides :

4x² + 4x - 8 = 0

Divide both sides by 4 :

x² + x - 2 = 0

Factorise using factorisation method :

x² - x + 2x -2 = 0

x(x - 1) +2(x-1) = 0

(x+2)(x-1) = 0

Solve for x

x +2 = 0   OR     x - 1 = 0        

x = -2       OR     x = 1

Answer will be Option B and D

Hope this helped and have a good day

Answer:

option b. –2 and d. 1 are correct.

*For detailed explanation refer to the attachment

Inventory was taken on Day 1, Day 2, Day 5, Day 6, and Day 7. Norris wants to know roughly what the inventory was on Day 3.

What should he do to estimate the inventory on Day 3?


He should use the value for Day 2 because it is closest in value to Day 3.


He should randomly pick a point near the fitted line where x=3.


He should evaluate the function f(x)=−1/3 x+4 for y=3.


He should evaluate the function f(x)=−1/3 x+4 for x=3.

Answers

Answer:

A

Step-by-step explanation:

guessing

He should evaluate the function f(x)=−1/3 x+4 for x=3  , Option D is the correct answer.

What in Interpolation ?

Interpolation is when the line of best fit is used to determine the value of a point that is within the range of plotted points.

It is given to find the value at Day 3 which lies in the range of the points used for plotting

The line for best fit is

f(x) = (-1/3)x +4

At x = 3

f(3) = (-1/3) * 3 +4

f(3) = -1+4 = 3

Therefore He should evaluate the function f(x)=−1/3 x+4 for x=3

Option D is the correct answer.

To know more about Interpolation

https://brainly.com/question/18768845

#SPJ1

Laurie is trying to stay within 10 feet of her current diving depth of –30 feet (with regard to sea level) so that the light is still good but she can be close to the sea life during her scuba dive. Which two equations can be used to find the minimum and maximum depths Laurie wants to stay between?

-30 - x = 10 and -30 - x = –10
-30 + x = 10 and -30 + x = –10
x + 10 = 30 and x + 10 = –30
x – 10 = 30 and x – 10 = –30

Answers

The equation that can be used to describe the minimum and maximum depths Laurie wants to stay between is x + 10 = 30 and x + 10 = –30 so the correct answer is option C.

What is an expression?

Expression in maths is defined as the collection of the numbers variables and functions by using signs like addition, subtraction, multiplication, and division.

Diving is the art of going down to the depth of the ocean. Divers usually go deep down in the ocean to study the dynamics of ocean life.

The equation that can be used to describe the minimum and maximum depths Laurie wants to stay between is x + 10 = 30 and x + 10 = –30

Therefore the equation that can be used to describe the minimum and maximum depths Laurie wants to stay between is x + 10 = 30 and x + 10 = –30 so the correct answer is option C.

To know more about Expression follow

https://brainly.com/question/723406

#SPJ1

its math help me out?

Answers

Its the first one

--------------------

Answer:

  $24 = $0.40(60)

Step-by-step explanation:

Match the input value and its location in the equation.

__

  $24 = $0.40(60)

_____

Additional comment

When input is liters and output is dollars, the constant of proportionality must have units of "dollars per liter." The dollar sign of these units is not shown in the left panel, but is shown on the answer choices. If you understand units conversion, this should not be a mystery. (The mystery is why the curriculum materials are inconsistent.)

Which expression can be used to find the difference of the polynomials? (10m – 6) – (7m – 4) [10m (–7m)] [(–6) 4] (10m 7m) [(–6) (–4)] [(–10m) (–7m)] (6 4) [10m (–7m)] [6 (–4)]

Answers

The expression can be used to find the difference of the polynomials is [10m (–7m)] [(–6) 4]

Difference of polynomials

A polynomial is a function that has a leading degree of 3 and above.

Given the expression below

(10m – 6) – (7m – 4)

Expand

10m - 6 - 7m  + 4

Collect the like terms

10m - 7m - 6 + 4

Simplify

3m -2

Hence the expression can be used to find the difference of the polynomials is [10m (–7m)] [(–6) 4]

learn more on difference of polynomial here: https://brainly.com/question/2516456

#SPJ1

The temperature fell at a rate of 0.65 °C/h. The temperature was recorded at 37 °C
at 6 p.m. Which function can be used to represent this situation?
Of(x) = 37 -0.65x
Of(x) = 0.65x - 37
Of(x) = 37x + 0.65
Of(x) = 0.65x+37

Answers

The linear function that can be used to represent the temperature in x hours after 6 pm is given by:

f(x) = 37 - 0.65x.

What is a linear function?

A linear function is modeled by:

y = mx + b

In which:

m is the slope, which is the rate of change, that is, by how much y changes when x changes by 1.b is the y-intercept, which is the value of y when x = 0, and can also be interpreted as the initial value of the function.

In this problem:

The y-intercept is the initial temperature of 37ºC.The slope is the rate of change of -0.65ºC/h.

Hence the function is:

f(x) = 37 - 0.65x.

More can be learned about linear functions at https://brainly.com/question/24808124

#SPJ1

Factor x² - 4x + 5.

A Prime
B (x + 5)(x - 1)
C (x - 5)(x - 1)
D (x + 5)(x + 1)

Answers

Answer:

[tex]\huge\boxed{\sf Option \ A}[/tex]

Step-by-step explanation:

Given expression:

= x² - 4x + 5

We can not factor out this expression, because of we apply mid-term break to it, the factors -4x will break into are -5x + x the coefficients of which when multiplied give -5 and not +5 (side term). This is a known rule in the mid-term break formula.

So, the given expression is a prime expression.

[tex]\rule[225]{225}{2}[/tex]

what does 5r-r-9=15 look like after combining like terms ?
and whats the answer for r?

Answers

The value of r from the expression is 6

Simplifying expression

Given the expression below;

5r-r-9=15

Add 9 to both sides

5r - r - 9 + 9 = 15 + 9

4r = 15 + 9

4r = 24

After combining the like terms the expression will be 4r = 24

4r/4 = 24/4

r = 6

Hence the value of r from the expression is 6

Learn more on expression here: https://brainly.com/question/723406

#SPJ1

Need Help Fast!!!!!! The graph of the piecewise function f(x) is shown. f(x) What is the range of f(x)?

Answers

Answer:

The second option

Step-by-step explanation:

If you look at the graph, it appears that from negative infinity to 0, the line is just constant, so the range of that would simply be the constant value or in this case 4. from 0 to infinity it appears the line is decreasing at a constant rate and should go towards negative infinity as x goes towards infinity. So the range would be -infinity < f(x) <= 4

HELP MEEEEEEEEEEEEEEEEEEEEEEEEEEEEEEEEEEEEE
write and solve a proportion to complete the statement. Round to the nearest hundredth if necessary.
1. 6km ≈ ?mi
2. 2.5 L≈ ?gal
3. 90lb ≈ ?kg
pls help

Answers

Using proportion, the equivalent units of the following units rounded to the nearest hundredth are as follows:

6km =  3.73 miles2.5 Litres = 0.66 gallons90 pounds = 40.82 kg

How to use proportion to find equivalent units?

Using proportion,

1.60934 km = 1 miles

6 km = ?

cross multiply

distance(m) = 6 / 1.60934 = 3.7282364199 = 3.73 miles

1 litre = 0.264172 gallons

2.5 litres = ?

volume(gallons) = 2.5 × 0.264172 = 0.66 gallons

1 pounds = 0.453592 kg

90 pounds  = ?

cross multiply

weight(kg) = 40.82328 = 40.82 kg

learn more on proportion here; brainly.com/question/14063625

#SPJ1

Which expression has a value of 16 when n = 5?
StartFraction 25 Over n EndFraction + 7
30 minus 3 n
7 + StartFraction 45 Over n EndFraction
n cubed minus 114

Answers

An expression is defined as a set of numbers, variables, and mathematical operations. The correct option is C.

What is an Expression?

In mathematics, an expression is defined as a set of numbers, variables, and mathematical operations formed according to rules dependent on the context.

To solve the problem substitute the value of n as 5 in each expression and then simplify to check if it equals to 16 or not.

A.) (25/n) + 7

      = (25/5) + 7

      = 12

This is not the required expression.

B.) 30 - 3n

      = 30 - 3(5)

      = 15

This is not the required expression.

C.) 7+ (45/n)

       = 7 + (45/5)

       = 7 + 9

       = 16

This is the required expression.

D.) n² - 114

       = (5)³ - 114

       = 125 - 114

       = 11

This is not the required expression.

Hence, the correct option is C.

Learn more about Expression:

https://brainly.com/question/13947055

#SPJ1

Consider the systems of equations below. determine the number of real solutions for each system of equations. system a has real solutions. system b has real solutions. system c has real solutions.

Answers

System A has 2 real solutions, System B has 0 real solutions and System C has 1 real solution.

Given a system of equations for A is x²+y²=17 and y=-(1÷2)x, a system of equations for B is y=x²-7x10 and y=-6x+5 and a system of equations for C is y=-2x²+9 and 8x-y=-17.

For system A,

The two systems of equations are

x²+y²=17            ......(1)

y=-1÷2x             ......(2)

Substitute the value of equation (2) into equation (1) as

x²+(-x÷2)²=17

x²+(x²÷4)=17

Simplify the above equation by taking L.C.M. as

(4x²+x²)÷4=17

5x²=68

x²=68÷5

x=±3.688

Find the value of y by substituting the value of x in equation (2).

When x=3.688 then y is

y=-(1÷2)×3.688

y=-1.844

And When x=-3.688 then y is

y=-(1÷2)×(-3.688)

y=1.844

Thus, the points where the equations of system A intersect each other is (3.688,-1.844) and (-3.688,1.844)

So, the system of equations of A has 2 real solutions.

For system B,

The two systems of equations are

y=x²-7x+10            ......(3)

y=-6x+5                ......(4)

Substitute the value of equation (4) into equation (3) as

-6x+5=x²-7x+10

x²-7x+10+6x-5=0

x²-x+5=0

Simplify the above quadratic equation using the discriminant rule,

x=(-b±√(b²-4ac))÷(2a)

Here, a=1, b=-1 and c=5

Substitute the values in the discriminant rule as

x=(1±√(1-4\times 5\times 1))÷2

x=(1±√(-19))÷2

x=(1±√(19)i)÷2

Here, the value of x goes into the complex.

So, the system of equations of B has 0 real solutions.

For system C,

The two systems of equations are

y=-2x²+9            ......(5)

8x-y=-17            ......(6)

Substitute the value of equation (6) into equation (5) as

8x-(-2x²+9)=-17

8x+2x²-9+17=0

2x²+8x+8=0

Simplify the above quadratic equation using factorization method as

2x²+4x+4x+8=0

2x(x+2)+4(x+2)=0

(2x+4)(x+2)=0

x=-2,-2

Find the value of y by substituting the value of x in equation (5).

When x=-2 then y is

y=-2(-2)²+9

y=-8+9

y=1

Thus, the point where the equations of system C intersect each other is (-2,1)

So, the system of equations of C has 1 real solutions.

Hence, the system of equations for A is x²+y²=17 and y=-(1÷2)x having 2 real solution, a system of equations for B is y=x²-7x10 and y=-6x+5 having 0 real solution and a system of equations for C is y=-2x²+9 and 8x-y=-17 having 1 real solution.

Learn about system of equations from here brainly.com/question/12962074

#SPJ4

A passenger jet plane cruises at 550 knots. It enters a jet stream with
a tailwind speed of 120 knots. If the speed of sound is 661 knots,
will the jet travel faster or slower than sound during its journey?

Answers

Considering the direction of the wind, it is found that the jet travels faster than the sound during it's journey.

What is the ground speed?

The jet's speed, considering that there is a tailwind, is given by:

J = Plane speed + Wind speed.

In this problem, we have that the speeds are given as follows:

Plane: 550 knots.Wind: 120 knots.

Hence the jet's speed is given by:

J = 550 + 120 = 670 knots.

670 knots > 661 knots, hence the jet travels faster than the sound during it's journey.

A similar problem, involving plane and wind's speed, is given at brainly.com/question/25547425

#SPJ1

Which expression is equivalent to startfraction (4 p superscript negative 4 baseline q) superscript negative 2 baseline over 10 p q superscript negative 3 baseline endfraction? assume p not-equals 0, q not-equals 0.

Answers

The given expression is equivalent to [tex]\frac{p^{7}q}{160}[/tex]

What are indices?

An index is a small number that tells us how many times a term has been multiplied by itself.

The plural of index is indices.

Below is an example of a term written in index form :[tex]4^{3}[/tex]

4 is the base and 3 is the index.

We can read this as ‘4 to the power 3’

Another way of expressing [tex]4^{3}[/tex] is

4 x 4 x 4 = 64

Indices can be positive or negative numbers.

Given expression can be written as [tex]\frac{({4p^{-4}q})^{-2}}{10pq^{-3}}[/tex]

Now to simplify the given fractional expression :[tex]\frac{({4p^{-4}q})^{-2}}{10pq^{-3}}[/tex]

=   [tex]\frac{4^{-2}p^{8}q^{-2}}{10pq^{-3}}[/tex]                    By using the property of exponents is given by:

                                    [tex](a^{m})^{n}=a^{m n}[/tex]

=[tex]\frac{p^{7}q}{10 .16}[/tex]                              By using the property of exponents given by

                                       [tex]a^{m}a^{n}=a^{m + n}[/tex]  and  [tex]a^{-m}= \frac{1}{a^{m}}[/tex]

= [tex]\frac{p^{7}q}{160}[/tex]

Learn about indices here :

https://brainly.com/question/27327380

#SPJ4

Mr. Lopez fruit salad recipe requires 3/4 of a cup of fresh peaches for 1 serving. He uses 9 cups of fresh peaches to prepare the salad. How many servings of the fruit salad did he prepare?

Answers

Answer:

6.75 servings

Step-by-step explanation:

[tex] \frac{3}{4} \times 9 = \frac{27}{4} = 6 \frac{3}{4} = 6.75[/tex]

Answer:

9 servings

Step-by-step explanation:

feel free to ask where you don't understand.

Each of 36 students at a school play bought either a cup of orange juice or a sandwich. A cup of orange juice costs $1 and a sandwich costs $3. The total amount collected was $76. How many students bought orange juice, and how many bought a sandwich?

Let represent the number of students who bought a cup of orange juice and represent the number of students who bought a sandwich. Then the problem can be represented by this system of equations:

+ 3 = 76
+ = 36

Answer the questions to solve the problem.

1. Explain what you should do with the two equations to eliminate one of the variables.

Answers

In order to eliminate one of the variables, subtract one of the equation from the other equation.

How to eliminate one of the variables?

Given these equations:

o + 3s = 76 equation 1

o + s = 36 equation 2

Where:

o = number of orange juice bought

s = number of sandwiches bought

In order to eliminate one of the variables, subtract equation 2 from equation 1. The result is :

2s = 40

s = 20

To learn more about simultaneous equations, please check: https://brainly.com/question/25875552

#SPJ1

Ahman has a lawn care business. he charges $25 per lawn to mow the grass. if his monthly expenses are $100, how many lawns must he mow in order to make a profit of at least $250 per month?

Answers

Answer:

14

Step-by-step explanation:

you can write an equation to represent the situation:

let x represent the number of lawns he has to mow

then, the monthly profit would be 25x - 100.

in order to make $250, this equation has to be equal to 250:

[tex]25x-100=250[/tex]

now, solve this:

add 100 to both sides

[tex]25x = 350[/tex]

divide both sides by 25

[tex]x = 14[/tex]

he must mow 14 lawns

sum of numbers 975,983,923,913 and 985 rounded upto hundredth place is

Answers

4,779 if you are just adding to find the sun of the numbers

Answer:

4800

Step-by-step explanation:

If a question is asking for the "sum" of numbers, this just means we have to add them altogether.

975+983+923+913+985 = 4779

To find the hundredth place, we use our place value. 4 is in the thousands column, and 7 is in the hundreds. Once we find the hundreds, we need the number to the right of it (7).

7 > 5 so we round up. 7 becomes 8.

4800.

Work out, giving your answer in its simplest form:
2/3 x 3 3/5

Answers

Answer: 12/5

Step-by-step explanation:

[tex]3 \frac{3}{5}=\frac{18}{5}\\\\\implies \frac{2}{3} \times 3 \frac{3}{5}=\frac{2}{3} \times \frac{18}{5}\\\\=\frac{36}{15}\\\\=\boxed{\frac{12}{5}}[/tex]

8 1/2 - 3/8=?
pls answer fast ​

Answers

Answer:

65/8

Step-by-step explanation:

1) Turn all numbers into improper fractions: 17/2 - 3/8 = ?

2) Make all denominators the same number by finding the least common factor, which is 8. Multiply the denominator in 17/2, which is 2, by 4 to match the other denominator. And then multiply the numerator (17) by 4 as well so that the fraction still has the same value: 68/8 - 3/8 = 65/8

3) Can not simplify since there are no common factors between 65 and 8.

Select the correct answer.
Which value of n makes the equation true?
-1/2n=-8
OA. -16
OB. - -4
OC. 4
O D. 16

Answers

ответ: д
-1/2 * 16 = -8
или 16/(-2) = -8

Answer: by rewriting equation in the form, / 1\2 16 X 4X 4 f=~1_~) (Do-2)2=g=~

Step-by-step explanation: hope this helps

Which expression is the simplest form of 2x^3 - x^2 + 3 (x^3 - 4x^2)

Answers

The simplest form of the given expression is 5x^3-5x^2.

We have given that

[tex]2x^3 - x^2 + 3 (x^3 - 4x^2)[/tex]

We have to determine the simplest form of the given expression.

What is the distributive property?

The distributive property of binary operations generalizes the distributive law, which asserts that equality is always true in algebra. elementary.

Use the distributive property we get,

[tex]2x^3 - x^2 + 3 (x^3 - 4x^2)\\=2x^3 - x^2 +3x^3-12x^2\\[/tex]

Add like terms we get,

Therefore we get,

[tex]=5x^3-5x^2[/tex]

Therefore the simplest form of the given expression is 5x^3-5x^2.

To learn more about the  expression visit:

https://brainly.com/question/723406

#SPJ1

CAN SOMEONE HELP ME PLS?????

Answers

You are correct, P(red) = P(green) = 0.25.

Select all the functions that show an inverse variation...
a) y=6x
b) y=3/x
c) y=1/3x
d) y= -.07x
e) y= 2/x+4
f) xy=-3
PLEASE HELP ILL MARK BRAINLEST

Answers

A and d would be the answers for this question

Find the slope of every line that is parallel to line on the graph (0,-3) (5,-4)

Answers

The slope of the line parallel to the line on the graph (0,-3) (5,-4) is - 1 / 5

How to find the slope of a parallel lines?

Parallel lines have the same slope.

Therefore,

slope = m = y₂ - y₁ / x₂ - x₁

Therefore,

x₁ = 0

x₂ = 5

y₁ = -3

y₂ = -4

Therefore,

slope = -4 - (-3)  / 5 - 0

slope = -4 + 3 / 5

slope = - 1 / 5

learn more on slope here: https://brainly.com/question/8978282

#SPJ1

(02.07)
The equation below shows the relationship between the
temperature in degrees Celsius, C, and degrees Fahrenheit, F:
H
C
(F-32)
Which of the following formulas correctly solves for F? (1 point)

Answers

Answer:

[tex]\boxed {\frac{F -32}{9} = \frac{C }{5}}[/tex]

Step-by-step explanation:

The correct relation between degrees Celsius, °C, and degrees Fahrenheit, °F is :

[tex]\boxed {\frac{F -32}{9} = \frac{C }{5}}[/tex]

$Need help with this pls (quickest answer gets brainliest)

Answers

Answer:

-1/2

Step-by-step explanation:

IK

John is 1/8 meter shorter than Paul, and Paul is 0.25 meter taller than Andrew. John's height is 13/4 meters, what is Andrews height?

Answers

Answer:

About 2.13

Step-by-step explanation:

13/4 multiply by 7/8 to get 91/32, then multiply by 0.75(3/4), which is 273/128

The weight of a cat is normally distributed with a mean of 9 pounds and a standard deviation of 2 pounds. Using the empirical rule, what is the probability that a cat will weigh less than 11 pounds?

Answers

If the value of the z-score is 1. Then the probability that a cat will weigh less than 11 pounds will be 0.84134.

What is the z-score?

The z-score is a statistical evaluation of a value's correlation to the mean of a collection of values, expressed in terms of standard deviation.

The z-score is given as

z = (x - μ) / σ

Where μ is the mean, σ is the standard deviation, and x is the sample.

The weight of a cat is normally distributed with a mean of 9 pounds and a standard deviation of 2 pounds.

Then the probability that a cat will weigh less than 11 pounds will be

The value of z-score will be

z = (11 – 9) / 2

z = 1

Then the probability will be

P(x < 11) = P(z < 1)

P(x < 11) = 0.84134

Thus, the probability that a cat will weigh less than 11 pounds will be 0.84134.

More about the z-score link is given below.

https://brainly.com/question/15016913

#SPJ1

Other Questions
Pick each reason that speakers use rhetorical devices:A:to help them influence their audience B:to emphasize important points C:to make a speech longer and seem more formalD:to help audience remember statistics or evidence related to the messageE:to ask the audience to determine the message of the speech on their ownF:to clarify information or important pointsG:to present information creatively and keep audience interested An event manager recorded the number of people in different age groups who attended a music concert:A histogram titled Concert Audience is shown. The horizontal axis is labeled Age Group in years with bins 18 to 24, 25 to 31, 32 to 38, and 39 to 45. The vertical axis labeled Number of People with values from 0 to 120 at intervals of 20. The first bin goes to 80, the second goes to 120, the third goes to 40, and the last goes to 20.Which data table accurately represents the data in the histogram?A:Age Group Number of People1824 802531 1203238 403945 20B:Age Group Number of People1824 802531 2003238 2403945 260C:Age Group Number of People1824 202531 403238 1203945 80D:Age Group Number of People1824 2602531 2403238 2003945 80 16, 13, 10 is this sequence arithmetic or geometric and determine the common difference/ ratio in simplest form Why should you learn the meaning of word elements Imagine that youre listening to a classmate who is running for president of the student council. What is the most convincing support for why you should vote for this classmate?a story about the classmates friendshipsexamples of the classmates leadership experiencean explanation of why the classmate wants the positiona description of the classmates college plans If a = 6, then the value of b is... As the story opens, Paul is most upset by memories of --Billy Boy's deathhis parentshis training in dealing with couragean old song Which brain regions were removed during H.M.s surgery? Select all that apply- amygdala-hippocampus-medial temporal lobe-cerebellum-basal ganglia What is the solution to the equation 5x + 2(x-4)= 5x + x -10?03020-20-3 Use the terms from this unit as effectively as you can. Be sure to answer all of the questions, above, that are bold.Harlem-What Happens to a Dream Deferred?What happens to a dream deferred?Does it dry uplike a raisin in the sun?Or fester like a sore---and then run?Does it stink like rotten meat?Or crust and sugar over--like a syrupy sweet?Maybe it just sagslike a heavy load.Or does it explode? Marcus and his classmates are discussing i novel in class. Which discussion observations most suggest thatMarcus has prepared before presenting his point of view to the group? Select two options.D Marcus has strong opinions about the novel.O Marcus is polite and waits for his turn to speak.O Marcus has brought the novelto use as a reference.0 Marcus often agrees with other students' viewpoints.O Marcus supports his opinions with evidence from the text.Mark this and returnSave and ExitNextSubmit Planes X and Y and points J, K, L, M, and N areshown.XLKMYExactly how many planes contain points J, K, and N?0000O1O23 HELP !!! A taxi driver uses this formula to work out the price of a journey, in pounds. price of journey = 3 + 2 x distance in miles. Leah has 5, the journey to her home is 2.5 miles. She asks the taxi driver to take her as near to home as possible. How far will she need to walk to arrive home? On January 1, 2020, Archer, Incorporated, paid $100,000 for a 30% interest in Harley Corporation. This investee had assets with a book value of $550,000 and liabilities of $300,000. A patent held by Harley having a book value of $10,000 was actually worth $40,000 with a six-year remaining life. Any goodwill associated with this acquisition is considered to have an indefinite life. During 2020, Harley reported net income of $50,000 and paid dividends of $20,000 while in 2021 it reported net income of $75,000 and dividends of $30,000. Assume Archer has the ability to significantly influence the operations of Harley. The amount allocated to goodwill at January 1, 2020, is Mary is preparing a purchase offer for her buyer client. It's a fairly standard offer: a little less than list price, with the seller paying a little toward the buyer's closing costs. Mary fills in the blanks on a standard form used in her brokerage firm based on the current situation. Is this okay Which of these are factors of this polynomial?-3x3 + 15x2 + 3x - 150 (x + 1)3x(3x - 1)(x - 5)-3(x + 5)(x - 1)00 Select the correct answer. Whitney wanted to paint her flowerpot with a different color. She decided to mix the colors blue and green to create a new col this new color be categorized as? Select the correct answer . Whitney wanted to paint her flowerpot with a different color . She decided to mix the colors blue and green to create a new col this new color be categorized as ? Which event was a consequence of the Enlightenment?A. Scientists began questioning the Catholic Church's traditional explanations of the natural worldB. French citizens rebelled against the French monarchy and establish a representative form of governmentC. Many christians broke away from the Catholic Church to join the protestant reformation D. Europeans began rejecting the social contract in favor of systems that offered individual freedom. Who was responsible for passing the Canadian bill of rights? The cost of painting a room consists of twoparts. One part is directly proportional tothe area and the other to the square of thelength of the room.